Đến nội dung

xuanhoan23112002 nội dung

Có 95 mục bởi xuanhoan23112002 (Tìm giới hạn từ 15-05-2020)



Sắp theo                Sắp xếp  

#709350 Đề thi vào 10 chuyên Lê Hồng Phong - Nam Định 2018 - 2019 - Toán Chuyên

Đã gửi bởi xuanhoan23112002 on 27-05-2018 - 10:49 trong Tài liệu - Đề thi

Câu 5:

a. Có $\frac{1}{2}\sqrt{(a+3b)(b+3a)}\leq \frac{a+3b+b+3a}{4}=a+b$ (bất đẳng thức AM-GM)

Từ giả thiết: $\sqrt{a}+\sqrt{b}=1$ 

Bình phương 2 vế ta có: $2\sqrt{ab}=1-a-b$ 

Hay $4ab=(1-a-b)^2$

Nên bất đẳng thức cần chứng minh tương đương với:

$3(a+b)^2+(1-a-b)^2\geq 2(a+b)$

$\Leftrightarrow (2a+2b-1)^2\geq 0$ (luôn đúng)

Vậy bất đẳng thức đã cho được chứng minh.




#712416 Đề thi IMO 2018

Đã gửi bởi xuanhoan23112002 on 12-07-2018 - 21:52 trong Thi HSG Quốc gia và Quốc tế

Kết quả chính thức IMO 2018



#709395 Đề thi vào trường chuyên Thái Bình năm 2019 (vòng 2)

Đã gửi bởi xuanhoan23112002 on 27-05-2018 - 20:34 trong Tài liệu - Đề thi

Câu 6:

Ta có:$\frac{1}{\sqrt{5a^2+2ab+2b^2}}=\frac{1}{\sqrt{(2a+b)^2+(a-b)^2}}\leq \frac{1}{2a+b}\leq \frac{1}{9}(\frac{2}{a}+\frac{1}{b})$ (bất đẳng thức Schwarz)

Chứng minh tương tự như trên ta có:

$P\leq \frac{1}{3a}+\frac{1}{3b}+\frac{1}{3c}$

Ta cũng có:$\frac{1}{a}+\frac{1}{b}+\frac{1}{c}\leq \sqrt{3(\frac{1}{a^2}+\frac{1}{b^2}+\frac{1}{c^2})}=\sqrt{3}$ (bất đẳng thức AM-GM)

Từ đó ta có: $P\leq \frac{\sqrt{3}}{3}$

Đẳng thức xảy ra $\Leftrightarrow a=b=c=\sqrt{3}$

Vậy MaxP = $\frac{\sqrt{3}}{3}\Leftrightarrow a=b=c=\sqrt{3}$.




#705302 ĐỀ THI OLYMPIC 30/4 NĂM 2018 THPT LHP TP.HCM - KHỐI 10

Đã gửi bởi xuanhoan23112002 on 08-04-2018 - 23:13 trong Thi HSG cấp Tỉnh, Thành phố. Olympic 30-4. Đề thi và kiểm tra đội tuyển các cấp.

Bài 4:

Theo đề bài ta có: $p^2 -p+1=x^3$(x là số tự nhiên,x>1)

Hay $p(p-1)=(x-1)(x^2+x+1)$

Do $p$ là số nguyên tố nên $x-1$ hoặc $x^2+x+1$ chia hết cho$ p$

Nếu $x-1$ chia hết cho $p$ thì $x-1\ge p,x^2+x+1<p$ (vô lí với x là số tự nhiên >1) 

Do đó $x^2+x+1$ chia hết cho $p$ nên $x^2+x+1=pk$ (k là số tự nhiên)

Ta xét$ k=1,2$ không thỏa mãn

Xét $k\ge 3$

Thay vào phương trình ta được:$ p-1=(x-1)k$ hay $p=(x-1)k+1$

Từ đó ta có: $x^2+x+1=(xk-k+1)k$

Hay $x^2+x(1-k^2)+k^2-k+1=0$

Đây là phương trình bậc 2 ẩn x, để phương trình có nghiệm tự nhiên thì $\delta =k^4-6k^2+4k-3$ phải là số chính phương

Ta có: $(k^2-3)^2\le k^4-6k^2+4k-3<(k^2-2)^2$

Từ đó ta tìm được k=3 ta tìm được x=7, p=19 là số nguyên tố

Vậy p=19 là số nguyên tố thỏa mãn đề bài




#709502 Bất đẳng thức chọn lọc ôn chuyên

Đã gửi bởi xuanhoan23112002 on 29-05-2018 - 15:47 trong Bất đẳng thức và cực trị

Bài 3: Theo giả thiết ta có $0\leq a, b, c\leq 4$ nên

$$(4-a)(4-b)(4-c) \geq 0$$

$\Leftrightarrow 64+4(ab+bc+ca) \geq abc+16(a+b+c)$

$\Leftrightarrow ab+bc+ca\geq 8+\frac{abc}{4}\geq 8$ 

Do đó ta có: $P=(a+b+c)^2-(ab+bc+ca)\leq 36-8=28$

Đẳng thức xảy ra $\Leftrightarrow (a, b, c)=(0, 2, 4)$ và các hoán vị của nó

Vậy $MaxP=28$ $\Leftrightarrow (a, b, c)=(0, 2, 4)$ và các hoán vị của nó




#710076 Đề thi tuyển sinh vào chuyên Lê Quý Đôn tỉnh Quảng Trị năm 2018-2019

Đã gửi bởi xuanhoan23112002 on 06-06-2018 - 07:55 trong Tài liệu - Đề thi

Câu 6:

Từ giả thiết kết hợp với công thức khai triển bậc 4: $(a+b)^4=a^4+4a^3b+6a^2b^2+4ab^3+b^4$ ta có:

$Q=(x+3-x)^4-4x(3-x)(x^2+(3-x)^2)=81-2(9-x^2-(3-x)^2)(x^2+(3-x)^2)=81+2(x^2+(3-x)^2)^2-18(x^2+(3-x)^2)=2(x^2+(3-x)^2-5)^2+2(x^2+(3-x)^2)+31\geq 10+31=41$

Đẳng thức xảy ra  $$\Leftrightarrow x^2+(3-x)^2=5\Leftrightarrow 2x^2-6x+4=0\Leftrightarrow x=1, x=2$$

Vậy $Min Q=41\Leftrightarrow x=1, x=2$




#709874 $a^2 + b^2 + c^2 + abc = 4$

Đã gửi bởi xuanhoan23112002 on 04-06-2018 - 07:45 trong Bất đẳng thức và cực trị

Bạn có thể tham khảo tại đây: http://diendantoanho...2b2c2abc-geq-4/

Đáp án: $Min P=2\Leftrightarrow (a,b,c)=(2,0,0)$ và các hoán vị của nó

             $Max P=3\Leftrightarrow (a,b,c)=(1,1,1)$




#709905 $a^2 + b^2 + c^2 + abc = 4$

Đã gửi bởi xuanhoan23112002 on 04-06-2018 - 12:38 trong Bất đẳng thức và cực trị

Tìm max: http://diendantoanho...-định-năm-2018/ (chỉ việc thay mỗi số 2 thành số 1 thôi a trình bày đầy đủ rồi)

Tìm min:

Nếu cả 3 số a, b, c đều > 2 hiển nhiên suy ra điều vô lí

Do đó ta giả sử: $c\leq 2$ nên $abc\leq 2ab$

$\Rightarrow 4=a^2+b^2+c^2+2abc\leq a^2+b^2+c^2+2ab=(a+b)^2+c^2\leq (a+b+c)^2$

$\Rightarrow a+b+c\geq 2$




#710107 Đề thi tuyển sinh vào lớp 10 chuyên toán tỉnh Kiên Giang

Đã gửi bởi xuanhoan23112002 on 06-06-2018 - 14:53 trong Tài liệu - Đề thi

Bài 7: Bài bất đẳng thức có vẻ dễ nhỉ

Ta có:

$\frac{x^2}{y+z}+\frac{y^2}{z+x}+\frac{z^2}{x+y}\geq \frac{(x+y+z)^2}{2(x+y+z)}= \frac{x+y+z}{2}=1$ ( bất đẳng thức Schwarz)

Đẳng thức xảy ra $\Leftrightarrow x=y=z=\frac{2}{3}$

Vậy ta có điều phải chứng minh.




#710186 Đề thi tuyển sinh vào lớp 10 THPT chuyên tỉnh Vĩnh Phúc năm học 2018-2019

Đã gửi bởi xuanhoan23112002 on 07-06-2018 - 11:41 trong Tài liệu - Đề thi

Câu 3 (phỏng theo lời giải của thầy Võ Quốc Bá Cẩn) 

Ta có: $\frac{a^2}{a^2+ab+b^2}+\frac{c^2}{c(a+b+c)}\geq \frac{(a+c)^2}{a^2+b^2+c^2+ab+bc+ca}$ (bất đẳng thức Schwarz)

Làm tương tự với 3 phân thức còn lại ta có:

$\frac{a^2}{a^2+ab+b^2}+\frac{b^2}{b^2+bc+c^2}+\frac{c^2}{c^2+ca+a^2}\geq 1$

Đẳng thức xảy ra $\Leftrightarrow a=b=c> 0$

Vậy ta có điều phải chứng minh.




#710048 Đề thi tuyển sinh vào lớp 10 THPT chuyên tỉnh Vĩnh Phúc năm học 2018-2019

Đã gửi bởi xuanhoan23112002 on 05-06-2018 - 19:45 trong Tài liệu - Đề thi

Câu 5:

Từ giả thiết ta có: $c=a+b-\sqrt{ab}$

$P=\frac{c^2}{ab}+\frac{c^2}{a^2+b^2}+\frac{\sqrt{ab}}{a+b}$

$P\geq c^2(\frac{1}{2ab}+\frac{1}{2ab}+\frac{1}{a^2+b^2})+\frac{\sqrt{ab}}{a+b}\geq \frac{9c^2}{(a+b)^2+2ab}+\frac{\sqrt{ab}}{a+b} \geq \frac{6(a+b-\sqrt{ab})^2}{(a+b)^2}+\frac{\sqrt{ab}}{a+b}=6-\frac{11\sqrt{ab}}{a+b}+\frac{6ab}{(a+b)^2}=6(\frac{\sqrt{ab}}{a+b}-\frac{1}{2})^2-\frac{5\sqrt{ab}}{a+b}+\frac{9}{2}$ $\geq -\frac{5}{2}+\frac{9}{2}=2$ ( theo các BĐT AM-GM và Schwarz)

 Đẳng thức xảy ra $\Leftrightarrow a=b=c>0$

Vậy $MinP=2\Leftrightarrow a=b=c>0$




#669686 Trong 100.000.000 số hạng đầu tiên của dãy Fibonacci, có tồn tại hay...

Đã gửi bởi xuanhoan23112002 on 24-01-2017 - 13:57 trong Tổ hợp và rời rạc

Ký hiệu $F_{n}$ là số hạng thứ $n$ của dãy Fibonacci

Chia mỗi số $F_{n}$ cho $10^4$ đc các số dư là $r_{n}$ (mod $10^4$) với $r_{i}=\overline{0,9999}$

Xét tất cả các cặp có thể có của các số nguyên từ $0$ đến $9999$ có $10^8$ cặp $(r_{0};r_{0});(r_{0};r_{1});(r_{1};r_{2});...$

Giả sử ko có cặp nào là $(0;0)$ thì các cặp trên phải có 2 cặp trùng nhau theo nguyên lý Dirichlet, gsử:

  $r_{n}=r_{p}$ và $r_{n+1}=r_{p+1}$ ($n>p$)

$\Rightarrow r_{n-1}=r_{p-1} \Rightarrow r_{n-2}=r_{p-2} \Rightarrow ... \Rightarrow r_{1}=r_{n-p+2}=r_{n-p+1} \Rightarrow r_{n-p+1}=r_{n-p+2} \Rightarrow r_{n-p}=0$

Vậy tồn tại $1$ số $F_{n}$ chia hết cho $10^4$ hay có tận cùng là $4$ chữ số $0$

Mình không hiểu bài này lắm, gửi lại cho mình đi.




#691945 $P=\frac{(x+1)^{2}(y+1)^{2}}{(x-...

Đã gửi bởi xuanhoan23112002 on 31-08-2017 - 13:28 trong Bất đẳng thức và cực trị

Bài này khá dễ giống đề thi lớp 10 ở VĨnh Phúc,. Bạn có thể tham khảo, hãy like cho mình nhé




#705303 Cần lắm một lời giải thích !

Đã gửi bởi xuanhoan23112002 on 08-04-2018 - 23:22 trong Hình học phẳng

Đó là tính chất góc định hướng được tạo bởi 4 điểm đồng viên




#705137 chứng minh

Đã gửi bởi xuanhoan23112002 on 06-04-2018 - 23:24 trong Số học

chứng minh rằng với mọi n$\epsilon$$\mathbb{Z}$ thì $n^{5}$ và n có chữ số tận cùng giống nhau 

n5 - n =n(n-1)(n+1)(n2+1)

Dễ dàng chứng minh n5-n chia hết cho 2

Với n chia hết cho 5, chia 5 dư 1 hoặc 4 thì n5-n chia hết cho 5

Với n chia 5 dư 2 hoặc 3 thì n2+1 chia hết cho 5

Như vậy ta có n5-n chia hết cho 5, n5-n chia hết cho 2 và gcd(5,2)=1

Nên n5-n chia hết cho 10

Hay n5 và n có chữ số tận cùng giống nhau




#704997 tìm giá trị của x để a nguyên

Đã gửi bởi xuanhoan23112002 on 05-04-2018 - 21:04 trong Đại số

PP làm: lấy tử chia mẫu được phần nguyên và phân số rồi sử dụng tính chia hết




#703960 2(ab+bc+ca)+$\frac{1}{ab}+\frac{1...

Đã gửi bởi xuanhoan23112002 on 19-03-2018 - 23:21 trong Bất đẳng thức và cực trị

Su dung pp pqr
Dat p=a+b+c=3

q=ab+bc+ca

r=abc,r<=1

BDT tuong duong 2q+3/r>=9

Hay 2qr+3>=9r

Ma q>=3*can(r)( do q^2>=3pr)

Dua ve bpt an r giai voi chu y r<=1




#704021 2(ab+bc+ca)+$\frac{1}{ab}+\frac{1...

Đã gửi bởi xuanhoan23112002 on 20-03-2018 - 22:08 trong Bất đẳng thức và cực trị

bạn bị ngược dấu hay sao ấy

nguoc dau cho nao




#705345 Cần lắm một lời giải thích !

Đã gửi bởi xuanhoan23112002 on 09-04-2018 - 20:48 trong Hình học phẳng

Ý mk hỏi [mod $\pi$] nghĩa là gì

góc định hướng giữa 2 đường thẳng hơn nhau 1 bội của π

 




#705432 $\frac{1}{a^2+2b^2+3}+\frac{1}...

Đã gửi bởi xuanhoan23112002 on 10-04-2018 - 21:59 trong Bất đẳng thức và cực trị

Từ giả thiết suy ra abc<=1

a2+2b2+3>=2ab+2b+2>0

$\frac{1}{a2+2b2+3}$<=$\frac{1}{2ab+2b+2}$

Làm tương tự như trên với các phân thức còn lại cùng với abc<=1 ta có điều phải chứng minh




#709371 CMR: $\sum \frac{ab+c^{2}}{a+b}...

Đã gửi bởi xuanhoan23112002 on 27-05-2018 - 17:05 trong Bất đẳng thức và cực trị

Ta có: $\sum \frac{ab+c^2}{a+b}+\sum c= \sum \frac{(c+a)(c+b)}{a+b}\geq 2(a+b+c)$ (bất đẳng thức AM-GM)

$\Rightarrow \sum \frac{ab+c^2}{a+b}\geq a+b+c$

Đẳng thức xảy ra $\Leftrightarrow a=b=c> 0$

Vậy bất đẳng thức được chứng minh.




#709289 Bất đẳng thức trong đề thi vào lớp 10 Nam Định năm 2018

Đã gửi bởi xuanhoan23112002 on 26-05-2018 - 15:38 trong Bất đẳng thức và cực trị

Cho a, b, c là các số thực dương thỏa mãn: $a^2+b^2+c^2+abc=4$. Chứng minh rằng:

$2a+b+c\leq \frac{9}{2}$




#706210 Chứng minh MB vuông góc MN khó

Đã gửi bởi xuanhoan23112002 on 17-04-2018 - 20:40 trong Hình học

Bài này có thể sử dụng tích vô hướng của lớp 10




#709806 Tìm GTLN của $2x^2-3xy-2y^2$

Đã gửi bởi xuanhoan23112002 on 03-06-2018 - 07:30 trong Bất đẳng thức và cực trị

Ta có:

$A-36a=(2-25a)x^2-(3+20a)xy-(2-40a)y^2$

Coi phương trình trên là phương trình bậc 2 ẩn x tìm giá trị của a sao cho phương trình có nghiệm kép tức là$\Delta =0$ (Chú ý: Tìm giá trị lớn nhất thì $A-36a$ mang dấu trừ của 1 bình phương đủ nên $2-25a<0, 2-40a<0$)

Từ đó tìm được: $a=\frac{1}{12 }$




#709817 Đề thi lớp 10 môn Toán vào Trường THPT Chuyên Lam Sơn

Đã gửi bởi xuanhoan23112002 on 03-06-2018 - 10:57 trong Tài liệu - Đề thi

Câu 5: Bất đẳng thức đã cho tương đương với:

$a^2b+b^2c+c^2a+ab^2+bc^2+ca^2> a^3+b^3+c^3+2abc$

$\Leftrightarrow a(b-c)^2+b^2(c+a-b)+c^2(a+b-c)> 0$ (luôn đúng đó a, b, c là độ dài 3 cạnh của 1 tam giác)